0 Daumen
753 Aufrufe

Hi,

Ich möchte zeigen, dass die Repunit Rn genau dann durch Rm teilbar ist, wenn m|n.

$$ {R}_{n}=\frac{{10}^{n}-1}{9} $$

mit n∈ℕ.

Nun habe ich versucht es zu beweisen, bin mir aber unsicher ob der Beweis richtig und in sich logisch und schlüssig ist:

$$\frac { { 10 }^{ n }-1 }{ 9 } \equiv 0\quad mod\quad \frac { { 10 }^{ m }-1 }{ 9 } \quad \\ Aus\quad { R }_{ m }|{ R }_{ n }\quad folgt\quad 9{ R }_{ m }|9{ R }_{ n }\\ \Rightarrow { 10 }^{ n }-1\equiv 0\quad mod\quad { 10 }^{ m }-1\\ \Rightarrow { 10 }^{ n }\equiv 1\quad mod\quad { 10 }^{ m }-1\\ Da\quad m|n\\ { ({ 10 }^{ m }) }^{ k }\equiv 1\quad mod\quad { 10 }^{ m }-1\quad mit\quad k=\frac { n }{ m } \\ Aus\quad x*y\quad mod\quad p\quad =\quad [x\quad mod\quad p\quad *\quad y\quad mod\quad p]\quad mod\quad p\\ und\quad { 10 }^{ m }-1\equiv 0\quad mod\quad { 10 }^{ m }-1\quad und\quad damit\quad auch\quad { 10 }^{ m }\equiv 1\quad mod\quad { 10 }^{ m }-1\quad folgt:\\ 1^{ k }\equiv 1\quad mod\quad { 10 }^{ m }-1$$

Wäre nett wenn sich das jemand mit mehr Erfahrung in diesem Bereich anschauen könnte ^^

Gruß
EmNero

Avatar von 6,0 k

1 Antwort

0 Daumen
 
Beste Antwort

Hallo EmNero,

der Beweis hat ein paar schwerwiegende Probleme.

1) Zeigst du nicht, dass aus dem einen das andere folgt sondern benutzt irgendwie gleichzeitig, dass \(R_m|R_n\) und \(m|n\) gilt. Somit verlierst du schon mal das Ziel aus den Augen.

2) \(10^m-1\) ist nicht unbedingt eine Primzahl.

Hier mal ein Vorschlag für den Beweis (bin bisschen eingerostet, deswegen ist das ganze eventuell ziemlich unelegant).

I) Wenn \(m|n\) gilt, also \(n = km\) für ein \(k\in \mathbb{N} \), dann substituiere (zur besseren Verständlichkeit) \(10^m=z\).

Dann hast du \( (z^k-1) = (z-1) \sum \limits_{i=0}^{k-1}z^i\) und somit \(R_m|R_n\).

II) Nehmen wir an, dass \(R_m|R_n\) gilt, was bedeutet, dass  \(10^m-1|10^n-1\).

Außerdem nehmen wir an, dass \(m \nmid n\), was bedeutet, dass \(a,b \in \mathbb{N}\) existieren mit \(n=am+b \) und \(0 < b <m \).

Betrachte nun:

$$ 10^n-1 = 10^{am+b}-1 = 10^b \cdot 10^{am} -1 = 10^b \cdot 10^{am} - 10^b + 10^b -1 \\ = 10^b(10^{am}-1)+(10^b-1) $$

Nach I) gilt \(10^m-1 | 10^{am}-1 \), allerdings erhalten wir durch \(10^m-1 \nmid 10^b-1 \) den Widerspruch zu \(10^m-1|10^n-1\). Somit muss also \(m |n\) gelten.

Gruß

Avatar von 23 k

Hi,

ich dachte in könnte die beiden Behauptungen verwenden und sie durch Umformung auf ein allgemein gültiges Ergebnis (nämlich 1^k kongruent 1 modulo (10^m)-1) bringen. Zeigt das nicht, dass beide Behauptungen wahr sein müssen? Sonst könnte ja am Ende kein wahres Ergebnis bei rauskommen, oder? Vielleicht vertue ich mich hierbei auch :D

Warum sollte 10m-1 eine Primzahl sein?

Deine Vorschläge in 2 muss ich mir mal in Ruhe anschauen.

Wikipedia meint es ist leicht zu zeigen, mehr dann aber auch nicht...

Für a<b

$$ { R }_{ ab }={ 10 }^{ a(b-1) }{ R }_{ a }+{ 10 }^{ a(b-2) }{ R }_{ a }+{ { 10 }^{ a(b-3) }R }_{ a }+...+{ { 10 }^{ a(b-(b-2)) }R }_{ a }+{ { 10 }^{ a(b-(b-1)) }R }_{ a }+{ R }_{ a }$$

Wäre ja dann durch Ra teilbar...

Gruß

Nein, dann zeigst du nicht die Äquivalenz \(A \Leftrightarrow B\) sondern, dass aus A und B irgendwas neues C folgt.

Weil \(10^m-1\) offensichtlich durch 9 teilbar ist ;) für \(m \geq 1\)

Das was du danach geschrieben hast steht bei mir unter I).

Nun ja ich habe doch aber nie behauptet, dass es eine Primzahl ist. Hmmm ich muss da erstmal wieder drüber nachdenken :D

Ah ok, war durch das \(p\) irritiert, aber stimmt das mit dem Produkt ist nicht auf Primzahlen beschränkt.

Was ist wenn ich den Beweis anders aufbaue:

$$ m|n\quad =>\quad k*m=n\\ \frac { { 10 }^{ n }-1 }{ 9 } \equiv x\quad mod\quad \frac { { 10 }^{ m }-1 }{ 9 } \\ \left( \frac { { 10 }^{ n }-1 }{ 9 } =t*\frac { { 10 }^{ m }-1 }{ 9 } +x\quad |*9\\ { 10 }^{ n }-1=t*{ 10 }^{ m }-1+9x \right) \\ { 10 }^{ n }-1\equiv 9x\quad mod\quad { 10 }^{ m }-1\\ { 10 }^{ n }\equiv 9x+1\quad mod\quad { 10 }^{ m }-1\\ { 10 }^{ km }\equiv 9x+1\quad mod\quad { 10 }^{ m }-1\\ Leicht\quad zu\quad zeigen:\\ { 10 }^{ m }\equiv 1\quad mod\quad { 10 }^{ m }-1\\ { 1 }^{ k }\equiv 9x+1\quad mod\quad { 10 }^{ m }-1\\ { 1 }^{ k }-1\equiv 9x\quad mod\quad { 10 }^{ m }-1\\ 0\equiv 9x\quad mod\quad { 10 }^{ m }-1\\ \Rightarrow x=0\\ \Rightarrow \frac { { 10 }^{ n }-1 }{ 9 } \equiv 0\quad mod\quad \frac { { 10 }^{ m }-1 }{ 9 } \\ etc. $$

Wenn das immer noch falsch ist probiere ich eben den anderen Weg :D

Ach sorry ich hatte dich ganz vergessen. Hoffe das kommt nicht allzu spät

Sieht gut aus. ;)

Zu spät kommt es auf keinen Fall, war ja nur eine Frage aus eigenem Interesse.

Aufjedenfall Danke für die Rückmeldung ^^

Mir fällt aber auf, dass ich auch etwas vergessen habe..

Ein anderes Problem?

Stell deine Frage

Willkommen bei der Mathelounge! Stell deine Frage einfach und kostenlos

x
Made by a lovely community